Difference between revisions of "1989 OIM Problems/Problem 2"

Line 1: Line 1:
 
== Problem ==
 
== Problem ==
Let <math>a</math>, <math>b</math>, and <math>c</math> be the longitudes of the sides of a triangle. Prove:
+
Let <math>x</math>, <math>y</math>, <math>z</math> three real numbers such that <math>0<x<y<z<\frac{\pi}{2}</math>. Prove the following inequality:
<cmath>\frac{a-b}{a+b}+\frac{b-c}{b+c}+\frac{c-a}{c+a}<\frac{1}{16}</cmath>
+
<cmath>\frac{\pi}{2}+2sin(x)cos(y)+2sin(y)cos(z)?sin(2x)+sin(2y)+sin(2z)</cmath>
  
 
~translated into English by Tomas Diaz. ~orders@tomasdiaz.com
 
~translated into English by Tomas Diaz. ~orders@tomasdiaz.com

Revision as of 13:18, 13 December 2023

Problem

Let $x$, $y$, $z$ three real numbers such that $0<x<y<z<\frac{\pi}{2}$. Prove the following inequality: \[\frac{\pi}{2}+2sin(x)cos(y)+2sin(y)cos(z)?sin(2x)+sin(2y)+sin(2z)\]

~translated into English by Tomas Diaz. ~orders@tomasdiaz.com

Solution

This problem needs a solution. If you have a solution for it, please help us out by adding it.